is it consistent with AC that every set is measurable? [duplicate]












-4












$begingroup$



This question already has an answer here:




  • Lebesgue-measurable sets requiring the Axiom of Choice to construct

    1 answer




Does every set is measurable follow from AC, or from it's negation?
I think that by Vitali's construction from the AC follows that some set is not masurable. But here in the 1st comment they claim that it may be consistent with ZFC that every set is measurable. How is it?










share|cite|improve this question











$endgroup$



marked as duplicate by Asaf Karagila axiom-of-choice
Users with the  axiom-of-choice badge can single-handedly close axiom-of-choice questions as duplicates and reopen them as needed.

StackExchange.ready(function() {
if (StackExchange.options.isMobile) return;

$('.dupe-hammer-message-hover:not(.hover-bound)').each(function() {
var $hover = $(this).addClass('hover-bound'),
$msg = $hover.siblings('.dupe-hammer-message');

$hover.hover(
function() {
$hover.showInfoMessage('', {
messageElement: $msg.clone().show(),
transient: false,
position: { my: 'bottom left', at: 'top center', offsetTop: -7 },
dismissable: false,
relativeToBody: true
});
},
function() {
StackExchange.helpers.removeMessages();
}
);
});
});
Dec 22 '18 at 20:36


This question has been asked before and already has an answer. If those answers do not fully address your question, please ask a new question.














  • 1




    $begingroup$
    Vitali constructs a not Lebesgue-measurable set with the axiom of choice.
    $endgroup$
    – Viktor Glombik
    Dec 22 '18 at 20:32








  • 3




    $begingroup$
    How could you first comment claim that it is consistent that every set is measurable with ZFC? It is exactly not what the comment says.
    $endgroup$
    – Asaf Karagila
    Dec 22 '18 at 20:37
















-4












$begingroup$



This question already has an answer here:




  • Lebesgue-measurable sets requiring the Axiom of Choice to construct

    1 answer




Does every set is measurable follow from AC, or from it's negation?
I think that by Vitali's construction from the AC follows that some set is not masurable. But here in the 1st comment they claim that it may be consistent with ZFC that every set is measurable. How is it?










share|cite|improve this question











$endgroup$



marked as duplicate by Asaf Karagila axiom-of-choice
Users with the  axiom-of-choice badge can single-handedly close axiom-of-choice questions as duplicates and reopen them as needed.

StackExchange.ready(function() {
if (StackExchange.options.isMobile) return;

$('.dupe-hammer-message-hover:not(.hover-bound)').each(function() {
var $hover = $(this).addClass('hover-bound'),
$msg = $hover.siblings('.dupe-hammer-message');

$hover.hover(
function() {
$hover.showInfoMessage('', {
messageElement: $msg.clone().show(),
transient: false,
position: { my: 'bottom left', at: 'top center', offsetTop: -7 },
dismissable: false,
relativeToBody: true
});
},
function() {
StackExchange.helpers.removeMessages();
}
);
});
});
Dec 22 '18 at 20:36


This question has been asked before and already has an answer. If those answers do not fully address your question, please ask a new question.














  • 1




    $begingroup$
    Vitali constructs a not Lebesgue-measurable set with the axiom of choice.
    $endgroup$
    – Viktor Glombik
    Dec 22 '18 at 20:32








  • 3




    $begingroup$
    How could you first comment claim that it is consistent that every set is measurable with ZFC? It is exactly not what the comment says.
    $endgroup$
    – Asaf Karagila
    Dec 22 '18 at 20:37














-4












-4








-4





$begingroup$



This question already has an answer here:




  • Lebesgue-measurable sets requiring the Axiom of Choice to construct

    1 answer




Does every set is measurable follow from AC, or from it's negation?
I think that by Vitali's construction from the AC follows that some set is not masurable. But here in the 1st comment they claim that it may be consistent with ZFC that every set is measurable. How is it?










share|cite|improve this question











$endgroup$





This question already has an answer here:




  • Lebesgue-measurable sets requiring the Axiom of Choice to construct

    1 answer




Does every set is measurable follow from AC, or from it's negation?
I think that by Vitali's construction from the AC follows that some set is not masurable. But here in the 1st comment they claim that it may be consistent with ZFC that every set is measurable. How is it?





This question already has an answer here:




  • Lebesgue-measurable sets requiring the Axiom of Choice to construct

    1 answer








set-theory axiom-of-choice measurable-functions






share|cite|improve this question















share|cite|improve this question













share|cite|improve this question




share|cite|improve this question








edited Dec 22 '18 at 22:13









Andrés E. Caicedo

66.1k8160252




66.1k8160252










asked Dec 22 '18 at 20:27









user122424user122424

1,1962717




1,1962717




marked as duplicate by Asaf Karagila axiom-of-choice
Users with the  axiom-of-choice badge can single-handedly close axiom-of-choice questions as duplicates and reopen them as needed.

StackExchange.ready(function() {
if (StackExchange.options.isMobile) return;

$('.dupe-hammer-message-hover:not(.hover-bound)').each(function() {
var $hover = $(this).addClass('hover-bound'),
$msg = $hover.siblings('.dupe-hammer-message');

$hover.hover(
function() {
$hover.showInfoMessage('', {
messageElement: $msg.clone().show(),
transient: false,
position: { my: 'bottom left', at: 'top center', offsetTop: -7 },
dismissable: false,
relativeToBody: true
});
},
function() {
StackExchange.helpers.removeMessages();
}
);
});
});
Dec 22 '18 at 20:36


This question has been asked before and already has an answer. If those answers do not fully address your question, please ask a new question.









marked as duplicate by Asaf Karagila axiom-of-choice
Users with the  axiom-of-choice badge can single-handedly close axiom-of-choice questions as duplicates and reopen them as needed.

StackExchange.ready(function() {
if (StackExchange.options.isMobile) return;

$('.dupe-hammer-message-hover:not(.hover-bound)').each(function() {
var $hover = $(this).addClass('hover-bound'),
$msg = $hover.siblings('.dupe-hammer-message');

$hover.hover(
function() {
$hover.showInfoMessage('', {
messageElement: $msg.clone().show(),
transient: false,
position: { my: 'bottom left', at: 'top center', offsetTop: -7 },
dismissable: false,
relativeToBody: true
});
},
function() {
StackExchange.helpers.removeMessages();
}
);
});
});
Dec 22 '18 at 20:36


This question has been asked before and already has an answer. If those answers do not fully address your question, please ask a new question.










  • 1




    $begingroup$
    Vitali constructs a not Lebesgue-measurable set with the axiom of choice.
    $endgroup$
    – Viktor Glombik
    Dec 22 '18 at 20:32








  • 3




    $begingroup$
    How could you first comment claim that it is consistent that every set is measurable with ZFC? It is exactly not what the comment says.
    $endgroup$
    – Asaf Karagila
    Dec 22 '18 at 20:37














  • 1




    $begingroup$
    Vitali constructs a not Lebesgue-measurable set with the axiom of choice.
    $endgroup$
    – Viktor Glombik
    Dec 22 '18 at 20:32








  • 3




    $begingroup$
    How could you first comment claim that it is consistent that every set is measurable with ZFC? It is exactly not what the comment says.
    $endgroup$
    – Asaf Karagila
    Dec 22 '18 at 20:37








1




1




$begingroup$
Vitali constructs a not Lebesgue-measurable set with the axiom of choice.
$endgroup$
– Viktor Glombik
Dec 22 '18 at 20:32






$begingroup$
Vitali constructs a not Lebesgue-measurable set with the axiom of choice.
$endgroup$
– Viktor Glombik
Dec 22 '18 at 20:32






3




3




$begingroup$
How could you first comment claim that it is consistent that every set is measurable with ZFC? It is exactly not what the comment says.
$endgroup$
– Asaf Karagila
Dec 22 '18 at 20:37




$begingroup$
How could you first comment claim that it is consistent that every set is measurable with ZFC? It is exactly not what the comment says.
$endgroup$
– Asaf Karagila
Dec 22 '18 at 20:37










1 Answer
1






active

oldest

votes


















5












$begingroup$

It doesn't follow from AC or from its negation.



In fact it is refuted by AC, as you pointed out, e.g. the Vitali set.



It doesn't follow from the negation, because a section of $mathbb{R}tomathbb{R/Q}$ is enough to perform the Vitali construction and certainly AC doesn't follow from such a section existing.



What is true is that under certain large cardinal hypotheses, it is consistent with ZF + Dependent choice that every set is measurable. And under no hypotheses at all it is consistent with ZF that every set is Borel (so measurable): indeed it is consistent with ZF that $mathbb{R}$ is a countable union of countable sets, and if this happens then so is every subset of $mathbb{R}$, and so every set is Borel as countable sets are Borel.



Finally, that's not what the linked 1st comment claims at all. It actually gives another proof that AC refutes said proposition : if every set were measurable, then the Banach Tarski paradox would actually be a paradox : it would be contradictory because the unit ball doesn't have measure $0$ or $infty$ and isometries preserve measure






share|cite|improve this answer









$endgroup$




















    1 Answer
    1






    active

    oldest

    votes








    1 Answer
    1






    active

    oldest

    votes









    active

    oldest

    votes






    active

    oldest

    votes









    5












    $begingroup$

    It doesn't follow from AC or from its negation.



    In fact it is refuted by AC, as you pointed out, e.g. the Vitali set.



    It doesn't follow from the negation, because a section of $mathbb{R}tomathbb{R/Q}$ is enough to perform the Vitali construction and certainly AC doesn't follow from such a section existing.



    What is true is that under certain large cardinal hypotheses, it is consistent with ZF + Dependent choice that every set is measurable. And under no hypotheses at all it is consistent with ZF that every set is Borel (so measurable): indeed it is consistent with ZF that $mathbb{R}$ is a countable union of countable sets, and if this happens then so is every subset of $mathbb{R}$, and so every set is Borel as countable sets are Borel.



    Finally, that's not what the linked 1st comment claims at all. It actually gives another proof that AC refutes said proposition : if every set were measurable, then the Banach Tarski paradox would actually be a paradox : it would be contradictory because the unit ball doesn't have measure $0$ or $infty$ and isometries preserve measure






    share|cite|improve this answer









    $endgroup$


















      5












      $begingroup$

      It doesn't follow from AC or from its negation.



      In fact it is refuted by AC, as you pointed out, e.g. the Vitali set.



      It doesn't follow from the negation, because a section of $mathbb{R}tomathbb{R/Q}$ is enough to perform the Vitali construction and certainly AC doesn't follow from such a section existing.



      What is true is that under certain large cardinal hypotheses, it is consistent with ZF + Dependent choice that every set is measurable. And under no hypotheses at all it is consistent with ZF that every set is Borel (so measurable): indeed it is consistent with ZF that $mathbb{R}$ is a countable union of countable sets, and if this happens then so is every subset of $mathbb{R}$, and so every set is Borel as countable sets are Borel.



      Finally, that's not what the linked 1st comment claims at all. It actually gives another proof that AC refutes said proposition : if every set were measurable, then the Banach Tarski paradox would actually be a paradox : it would be contradictory because the unit ball doesn't have measure $0$ or $infty$ and isometries preserve measure






      share|cite|improve this answer









      $endgroup$
















        5












        5








        5





        $begingroup$

        It doesn't follow from AC or from its negation.



        In fact it is refuted by AC, as you pointed out, e.g. the Vitali set.



        It doesn't follow from the negation, because a section of $mathbb{R}tomathbb{R/Q}$ is enough to perform the Vitali construction and certainly AC doesn't follow from such a section existing.



        What is true is that under certain large cardinal hypotheses, it is consistent with ZF + Dependent choice that every set is measurable. And under no hypotheses at all it is consistent with ZF that every set is Borel (so measurable): indeed it is consistent with ZF that $mathbb{R}$ is a countable union of countable sets, and if this happens then so is every subset of $mathbb{R}$, and so every set is Borel as countable sets are Borel.



        Finally, that's not what the linked 1st comment claims at all. It actually gives another proof that AC refutes said proposition : if every set were measurable, then the Banach Tarski paradox would actually be a paradox : it would be contradictory because the unit ball doesn't have measure $0$ or $infty$ and isometries preserve measure






        share|cite|improve this answer









        $endgroup$



        It doesn't follow from AC or from its negation.



        In fact it is refuted by AC, as you pointed out, e.g. the Vitali set.



        It doesn't follow from the negation, because a section of $mathbb{R}tomathbb{R/Q}$ is enough to perform the Vitali construction and certainly AC doesn't follow from such a section existing.



        What is true is that under certain large cardinal hypotheses, it is consistent with ZF + Dependent choice that every set is measurable. And under no hypotheses at all it is consistent with ZF that every set is Borel (so measurable): indeed it is consistent with ZF that $mathbb{R}$ is a countable union of countable sets, and if this happens then so is every subset of $mathbb{R}$, and so every set is Borel as countable sets are Borel.



        Finally, that's not what the linked 1st comment claims at all. It actually gives another proof that AC refutes said proposition : if every set were measurable, then the Banach Tarski paradox would actually be a paradox : it would be contradictory because the unit ball doesn't have measure $0$ or $infty$ and isometries preserve measure







        share|cite|improve this answer












        share|cite|improve this answer



        share|cite|improve this answer










        answered Dec 22 '18 at 20:36









        MaxMax

        16.3k11144




        16.3k11144















            Popular posts from this blog

            Plaza Victoria

            In PowerPoint, is there a keyboard shortcut for bulleted / numbered list?

            How to put 3 figures in Latex with 2 figures side by side and 1 below these side by side images but in...